I am having a hard time with this. I also chose B. I tried to work out the problem as Charlie explained where T receives the virus, and transmits it Q, which transmits it to R and S. However, S has to transmit it to one computer and we are left with U and P. P has to receive the virus from T or U, so S must transmit the virus to U, but now where does P go? What am I missing here that would resolve my issue? Otherwise, I do not see how Q would not be correct. I do see why R is correct, but I just can't seem to create a diagram to make Q incorrect. Below is my replication of the diagram described above.
--- R --- P (wrong)
Virus --- T -- Q
--- S --- U
Charlie Melman wrote:Hi Claudia,
Answer choice (B) is incorrect because there is one setup where Q pushes the virus to two other computers. If Q is the piece that transmits to R and S, and T is the piece that transmits to Q (as the rules allow), then it's entirely possible to construct a situation that does not violate any of the rules.
But in answer choice (C), you can't satisfy the second-to-last and last rules at the same time, while also making R transmit to two other computers. You can satisfy one of the two, but not both.
I hope this makes sense!